Last visit was: 25 Apr 2024, 19:55 It is currently 25 Apr 2024, 19:55

Close
GMAT Club Daily Prep
Thank you for using the timer - this advanced tool can estimate your performance and suggest more practice questions. We have subscribed you to Daily Prep Questions via email.

Customized
for You

we will pick new questions that match your level based on your Timer History

Track
Your Progress

every week, we’ll send you an estimated GMAT score based on your performance

Practice
Pays

we will pick new questions that match your level based on your Timer History
Not interested in getting valuable practice questions and articles delivered to your email? No problem, unsubscribe here.
Close
Request Expert Reply
Confirm Cancel
SORT BY:
Kudos
User avatar
Senior Manager
Senior Manager
Joined: 25 Jun 2011
Status:Finally Done. Admitted in Kellogg for 2015 intake
Posts: 396
Own Kudos [?]: 16655 [17]
Given Kudos: 217
Location: United Kingdom
Concentration: International Business, Strategy
GMAT 1: 730 Q49 V45
GPA: 2.9
WE:Information Technology (Consulting)
Send PM
Most Helpful Reply
Math Expert
Joined: 02 Sep 2009
Posts: 92915
Own Kudos [?]: 619040 [6]
Given Kudos: 81595
Send PM
General Discussion
Manager
Manager
Joined: 06 Jan 2012
Status:May The Force Be With Me (D-DAY 15 May 2012)
Posts: 165
Own Kudos [?]: 2095 [1]
Given Kudos: 33
Location: India
Concentration: General Management, Entrepreneurship
Send PM
avatar
Intern
Intern
Joined: 16 Feb 2012
Posts: 34
Own Kudos [?]: 13 [0]
Given Kudos: 2
Location: United States
Concentration: Entrepreneurship, Technology
GMAT 1: 690 Q47 V38
GPA: 3.7
Send PM
Re: Games won [#permalink]
enigma123 wrote:
After winning 80 percent of the fi…rst 40 games it played, Team A lost all the remaining games it played. How many total games did Team A play?
(1) Team A lost all of its games against Team B, which won 75 percent of its total games.
(2) Team lost exactly 1/3 of all the games it played.

Guys - I am consistently getting these type of questions wrong. Can someone please explain the step by step approach?


Even though I answered it wrong, I think I know why.

Team won 80% of first 40 games => Win = 32 Lost/ Tie 8 out of first 40. and lost all after wards. So total Win (32) + Lost (x) + Tie/ Lost (8)
1) Team B won 75% of games it played and Team A lost all against it. Do not know how many teams are there over all. How many games each team plays agains another.
2) Team lost 1/3 of the game. **HERE I MADE MISTAKE** lost =1/3 NOT EQUALs TO 2/3 won because there could be tie in some of the 8 games out of first 40. So, not sufficient.

Hence E.
User avatar
Manager
Manager
Joined: 12 Oct 2011
Posts: 100
Own Kudos [?]: 730 [0]
Given Kudos: 23
GMAT 1: 700 Q48 V37
GMAT 2: 720 Q48 V40
Send PM
Re: Games won [#permalink]
Tgt750 wrote:
enigma123 wrote:
After winning 80 percent of the fi…rst 40 games it played, Team A lost all the remaining games it played. How many total games did Team A play?
(1) Team A lost all of its games against Team B, which won 75 percent of its total games.
(2) Team lost exactly 1/3 of all the games it played.

Guys - I am consistently getting these type of questions wrong. Can someone please explain the step by step approach?


Even though I answered it wrong, I think I know why.

Team won 80% of first 40 games => Win = 32 Lost/ Tie 8 out of first 40. and lost all after wards. So total Win (32) + Lost (x) + Tie/ Lost (8)
1) Team B won 75% of games it played and Team A lost all against it. Do not know how many teams are there over all. How many games each team plays agains another.
2) Team lost 1/3 of the game. **HERE I MADE MISTAKE** lost =1/3 NOT EQUALs TO 2/3 won because there could be tie in some of the 8 games out of first 40. So, not sufficient.

Hence E.

I don't think the question assumes that ties are possible.
avatar
Manager
Manager
Joined: 12 Mar 2012
Posts: 66
Own Kudos [?]: 523 [0]
Given Kudos: 22
Location: India
Concentration: Technology, Strategy
GMAT 1: 710 Q49 V36
GPA: 3.2
WE:Information Technology (Computer Software)
Send PM
Re: After winning 80 percent of the fi…rst 40 games it played [#permalink]
Statement 2 is sufficient because we know how many it won and how many it lost.
Intern
Intern
Joined: 31 Mar 2021
Posts: 22
Own Kudos [?]: 4 [0]
Given Kudos: 291
Send PM
Re: After winning 80 percent of the fi…rst 40 games it played [#permalink]
Won-32 lost-8. Now, rest all loose then 8+X. Total-40+x. Stmt1 insufficient bcoz- no information about team B games.,….. stamnt-2 (40+x)/3=x+8. We can solve for x

Posted from my mobile device
User avatar
Non-Human User
Joined: 09 Sep 2013
Posts: 32680
Own Kudos [?]: 822 [0]
Given Kudos: 0
Send PM
Re: After winning 80 percent of the fi…rst 40 games it played [#permalink]
Hello from the GMAT Club BumpBot!

Thanks to another GMAT Club member, I have just discovered this valuable topic, yet it had no discussion for over a year. I am now bumping it up - doing my job. I think you may find it valuable (esp those replies with Kudos).

Want to see all other topics I dig out? Follow me (click follow button on profile). You will receive a summary of all topics I bump in your profile area as well as via email.
GMAT Club Bot
Re: After winning 80 percent of the fi…rst 40 games it played [#permalink]
Moderator:
Math Expert
92915 posts

Powered by phpBB © phpBB Group | Emoji artwork provided by EmojiOne